Maximum value for x

Algebra Level 4

If x x , y y and z z are positive real numbers such that x + y + z = 5 x+y+z=5 and x y + y z + z x = 3 xy+yz+zx=3 , then what is the largest possible value of x x ?


The answer is 4.333.

This section requires Javascript.
You are seeing this because something didn't load right. We suggest you, (a) try refreshing the page, (b) enabling javascript if it is disabled on your browser and, finally, (c) loading the non-javascript version of this page . We're sorry about the hassle.

2 solutions

Vilakshan Gupta
Sep 16, 2017

From the identity ( x + y + z ) 2 = x 2 + y 2 + z 2 + 2 ( x y + y z + z x ) (x+y+z)^2 = x^{2}+y^{2}+z^{2}+2(xy+yz+zx) , and substituting the values given , we get x 2 + y 2 + z 2 = 19 y 2 + z 2 = 19 x 2 A l s o , y + z = 5 x x^{2}+y^{2}+z^{2}=19 \\ \implies y^{2}+z^{2}=19-x^2 \\ Also, \hspace{0.25cm} y+z=5-x From Cauchy-Schwarz Inequality , y 2 + z 2 1 2 ( y + z ) 2 y^{2}+z^{2} \geq \frac{1}{2}(y+z)^{2} Putting the values of y 2 + z 2 y^{2}+z^{2} and y + z y+z , we get: 19 x 2 1 2 ( 5 x ) 2 19-x^2 \geq \frac{1}{2}(5-x)^2 On simplifying we are left with 3 x 2 10 x 13 0 3x^2-10x-13 \leq 0 which implies ( 3 x 13 ) ( x + 1 ) 0 (3x-13)(x+1) \leq 0 yielding maximum value as 13 3 \dfrac{13}{3} \approx 4.33 \boxed{4.33}


Note that for showing that the value is achievable, we substitute x = 13 3 x=\dfrac{13}{3} from which we get that y = z = 1 3 y=z=\dfrac 13 . Hence it is indeed achievable.

Nice solution. This problem also appears here , by the way.

James Wilson - 3 years, 4 months ago

Log in to reply

Also, I believe you must check to make sure x = 13 3 x=\frac{13}{3} is achievable.

James Wilson - 3 years, 4 months ago

Log in to reply

Sure, I will edit my solution.

Vilakshan Gupta - 3 years, 4 months ago

Thanks for showing me a calculus solution.

Vilakshan Gupta - 3 years, 4 months ago

Log in to reply

Not a problem, my good Sir.

James Wilson - 3 years, 4 months ago

Log in to reply

@James Wilson Please don't call me a sir as I am only 16 years old!

Vilakshan Gupta - 3 years, 4 months ago

Log in to reply

@Vilakshan Gupta I didn't look at your age to be honest. Your skills are very impressive for your age. I hope it serves you well.

James Wilson - 3 years, 4 months ago

Log in to reply

@James Wilson Thank you so much sir.

Vilakshan Gupta - 3 years, 4 months ago
Pavneet Oberoi
Sep 16, 2017

Here, x+y+z=5; xy+yz+zx=3; x^2+ y^2+z^2=(x+y+z)^2-2(xy+yz+zz)=25-6=19; x(y+z)+zx=3; Applying AM-GM x(y+z)+(z^2+y^2)/2≥3; 2x(5-x)+(19-x^2)≥6; -3x^2+10x+19-6≥0; 3x^2-10x-13≥0; (3x-13)(x+1)≥0; -1≤x≤13/3; Therefore maximum value for x is 13/3; which is achievable as putting this value is the set of equation will yield positive value for both Z and Y

Hi, Pavneet. I was just looking at your solution, and I just wanted to comment on your application of the AM-GM inequality. It shows x 13 3 x\leq\frac{13}{3} , but it does not show x = 13 3 x=\frac{13}{3} is the maximum. In order to do that, you simply need to show that the value is achievable using y = z = 1 3 y=z=\frac{1}{3} . Cheers, James.

James Wilson - 3 years, 4 months ago

Log in to reply

Thankyou.,answer edited.

Pavneet oberoi - 3 years, 2 months ago

Log in to reply

No problem

James Wilson - 3 years, 2 months ago

0 pending reports

×

Problem Loading...

Note Loading...

Set Loading...